Please someone help me out. I beggggg

Please Someone Help Me Out. I Beggggg

Answers

Answer 1

Answer:

1.  144  2. 16 3. 1   4. 3x-6  

Step-by-step explanation:

So think of this as a function in a function.  So you work from the inside to the outside.  So for problem 1, we start with f(4)) [you read it "f of 4"] so what is the solution when x = 4, since f(x) means the function of x so f(4) means 'the function of 4' inside f(x).  

Since f(x) = 3x then f(4) = 3(4) [notice how you substitute the 4 everywhere you see a letter x]

so f(4) = 12, now you work the next part h(f(4)) since f(4)=12 then h(12)

So take the h(x) function which is h(x) = [tex]x^{2}[/tex] then h(12) = [tex]12^{2}[/tex] so h(12) = 144


Related Questions

Can someone save this for me ?

Answers

Answer:

1. point it to 381.5

2. point it to 33.5

3. point it to 1,436

4. point it to 3,052.1

for pointing it to in terms of pi im confused about because ive tried to get the answer but can't

Step-by-step explanation:

Please hurry I’m having trouble on this question

Answers

Answer:

c

Step-by-step explanation:

Answer:

B. Y = 1.5x

Step-by-step explanation:

What is 43.6925 rounded to the nearest thousands place

Answers

Answer:

43.693

Step-by-step explanation:

Unit 4 Homework 2 Gina Wilson all things algebra,
Pls help!

Answers

What?!?!?! This doesn’t make any sense

Answer:

1. m∠1 = 48°

2. m∠1 = 129°

3. m∠1 = 37°

4. m∠1 = 88°

m∠2 = 42°

m∠3 = 113°

5. m∠1 = 72°

m∠2 = 35°

m∠3 = 24°

6. m∠1 = 128°

m∠2 = 52°

m∠3 = 47°

m∠4 = 133°

m∠5 = 81°

7. m∠1 = 41°

m∠2 = 85°

m∠3 = 95°

m∠4 = 85°

m∠5 = 36°

m∠6 = 49°

m∠7 = 106°

8. x = 12°

9. x = 9°

10. x = 5°

11. x = 7°

Step-by-step explanation:

1. 76° + 56° + m∠1 = 180°

∴ m∠1 = 180° - (76° + 56°) = 48°

m∠1 = 48°

2. m∠1 = 67° + 62° = 129°

m∠1 = 129°

3. m∠1 = 152° - 115° = 37°

m∠1 = 37°

4. m∠1 = 180° - (52° + 42°) = 88°

m∠1 = 88°

m∠2 = 42°

m∠3 = 180 - (42° + 25°) = 113°

m∠3 = 113°

5. m∠1 = 180° - 118° = 72°

m∠1 = 72°

m∠2 = 180° - 72° - 73° = 35°

m∠2 = 35°

m∠3 = 73° - 49° = 24°

m∠3 = 24°

6. m∠5 = 180° - m∠52° - m∠47° = 81°

m∠5 = 81°

m∠1 = 81° + 47° = 128°

m∠1 = 128°

m∠2 = 52°

m∠3 = 47°

m∠4 = 180° - 47° = 133°

m∠4 = 133°

7. m∠3 = 95°

m∠5 = 180° - 144° = 36°

m∠5 = 36°

m∠6 = 180° - 95° - 36° = 49°

m∠6 = 49°

m∠4 = 180° - 95° = 85°

m∠4 = 85°

m∠2 = 85°

m∠7 = 360° - (38° + 85° + (180° - 49°)) = 106°

m∠7 = 106°

m∠1 = 360° - (85° + 90° + 144°) = 41°

m∠1 = 41°

8. (10·x - 11)° + (3·x - 2)° + (3·x + 1)° = 16·x° - 12° = 180°

16·x = 180°+ 12° = 192°

x° = 192°/16 = 12°

x = 12°

9. (3·x - 5)° + (7·x + 5)° + 90 = 180°

10·x + 90° = 180°

10·x  = 180°- 90° = 90°

10·x = 90°

x = 90°/10 = 9°

x = 9°

10. 180 - 151 = 29°

(11·x - 1)° + (20·x - 3)° + 29° = 180°

31·x° + 25° = 180°

31·x°  = 180° - 25° = 155°

31·x°  = 155°

x = 155/31 = 5°

x = 5°

12. (4·x + 13) + (6·x + 2) + 180 - (14·x - 13)

208° - 4·x° = 180°

208° -  180° = 4·x°

28° = 4·x°

4·x° = 28°

x = 28°/4 = 7°

x = 7°.

FIND THE LENGTH OF SIDE “DE”. GIVING BRAINLIEST

Answers

8
Needs 20 characters

how many moles are in 24 grams of argon?

Answers

the answer would be 39.948 grams
i’m smart lol

help me out with this pls (multiple choice)

Answers

Answer:

B

Step-by-step explanation:

[tex] \tan( 30) = \frac{y}{7 } \\ y = \sqrt{147} = 7 \sqrt{3} \\ {x}^{2} = {7}^{2} + {(7 \sqrt{3}) }^{2} \\ {x}^{2} = 49 + 147 = 196 \\ x = \sqrt{196} = 14[/tex]

It’s kinda easy but I wanna make sure it’s right pls help if you can

Answers

Answer:

a. 9/10

Step-by-step explanation:

hopes this helps

The director of sales at the Shonda car company wants to examine the
relationship between the amount of money the company spends on marketing
and the number of cars sold. The scatter plot below shows this relationship. Which of the following equations most likely represents the line of best fit for the data shown in the scatter plot?

Answers

Answer:

Step-by-step explanation:

the answer is the 3rd one hope this helps:)

Find the value of X ?

Answers

Answer:

x = 19

Step-by-step explanation:

(10x - 23) and (7x + 34) are vertically opposite angles and congruent, so

10x - 23 = 7x + 34 (subtract 7x from both sides )

3x - 23 = 34 ( add 23 to both sides )

3x = 57 ( divide both sides by 3 )

x = 19

Sixty-four is the product of eight and the sum of a number and six.

Answers

Answer:

64 is the sum of the numbers

58 + 6 = 64

PLEASE MARK ME AS BRAINLIEST

13. The surveys of interest for a concert produced these data points about the relation between ticket

price and ticket sales

Ticket Price (S)

3

5

7

9

11

13

Ticket Sales (S) 1447 1353 1212 1025 790 509

(A) Find the quadratic equation that best models the data,

14a)

(B) Predict the ticket sales if the tickets are $8 each.

14b)

(C) What would the price of the ticket have been if there

is $1000 in ticket sales?

14c)

Answers

Answer:

1500.110 - 0.157x - 5.853x² ;

1124.262

Step-by-step explanation:

Given the data:

Price __ Sales

3 ____1447

5 ___ 1353

7 ____ 1212

9 ____1025

11 ____790

13 ____509

Usibg the quadratic regression calculator :

1500.110 - 0.157x - 5.853x²

If ticket price = $8 each

Ticket sales will be :

y = 1500.110 - 0.157(8)- 5.853(8^2)

= 1124.262

please help mee

[tex]7 - ( - 3)[/tex]

Answers

Answer:

10

Step-by-step explanation:

7-(-3)=7+3=10

Which ordered pair is in the solution set of the system of inequalities
shown in the accompanying graph?*

Answers

Answer:

(-4,1)

Step-by-step explanation:

(6,1)- incorrect

(0,4) -incorrect

Try out pairs that will be inside the darker shaded color.

Hope this helps!

Virginia needed some batteries for her flashlights. She bought 4 batteries for $6.80. At that rate, how much would 10 batteries cost?

Answers

Answer:

$17

Step-by-step explanation:

First find the cost of 1 battery.

$6.80 ÷ 4 = $1.70

Then multiply the cost of one battery by 10.

$1.70 x 10 = $17

Answer:

10 batteries would cost $17

Cora is playing a game that involves flipping three coins at once let the random variable H be the number of coins that land showing heads. Here is the probability distribution for H.

Answers

Answer:

[tex]P(H <3) = 0.875[/tex]

Step-by-step explanation:

Given (Missing from the question):

[tex]\begin{array}{cc}{H}&{P(H)&0&0.125&1&0.375&2&0.375&3&0.125\end{array}[/tex]

And the required question is:

Determine the P(H < 3)

P(H <3) implies that: P(H = 0) or P(H = 1) or P(H = 2)

So, we have:

[tex]P(H <3) = P(H = 0) + P(H = 1) + P(H = 2)[/tex]

[tex]P(H <3) = 0.125 + 0.375 +0.375[/tex]

[tex]P(H <3) = 0.875[/tex]

Answer:

0.5

Step-by-step explanation:

That's the right answer on Khan

These are my cats, idek if it worked lol, also 40x2 bc why not you don’t have to answer






Answers

Answer:

It didn't work, 40 x 2 = 80

Step-by-step explanation:

have a good day

Answer:

Hey! I don't see any cats :(

Step-by-step explanation:

40x2= 80 !

Hope this helps!

Have a nice day! ♥(>'-'<)

PLEASE HELP ME, ITS ONLY ONE QUESTION!!!!!!!

Answers

Answer:

B and D are both correct.

B and C are correct

Suppose a rabbit population of 10 rabbits triples every month. Write and solve a function for how many rabbits will exist after 3 months.

Answers

Answer:

40

Step-by-step explanation:

10 rabbits

+ 3 months = 30 rabbits

40 rabbits

(5x-3)^2 -60x=(5x-3)^2

Answers

Answer:

x = 0

Step-by-step explanation:

Answer:

hope you can understand what im saying

can somebody help me please​

Answers

Answer:

x- 0,3 y-10,0

Step-by-step explanation:

Answers: (3,0)
(10,0)

Lydia wants to become a stuntwoman for the movies. She finds a great school online and gets accepted. The only
problem is that the school is in California and Lydia doesn't have a car or money for a plane ticket. It will cost her
$2,000 to buy a car that gets 30 miles per gallon. The trip is 1200 miles and gas is $4.00 per gallon. It is $2,500 for the
plane ticket. Which would be cheaper for her the car or the plane?

Answers

Answer:

The aeroplane would be cheaper.

Step-by-step explanation:

she only has to spend on the pane which would be $2,500.

It is coming around $2,800 (cost of car+gas bills) if she chooses to buy car.

Hope it helps!

6-Qual o resultado da expressão:129+3-12×3+15×6=
A) 77
B)97
C)90
D)87​

Answers

Answer:

1250

Step-by-step explanation:

that's what I got please let me know if I'm correct

zero point three divided by zero point 9

Answers

Answer:

0.33333

Step-by-step explanation:

Answer: the answer is 0.3 with a recipricol bar

Step-by-step explanation:

26 out of 65 as a percentage
How would you do that without a calculator?

Answers

Answer:

40%

Step-by-step explanation:

you would first simply the fraction down

13 goes into both numbers so the fraction can be simplified to

2/5 which is think is simply enough to know that 2/5 is 40% but if not the explanation would be

100÷5 =20

20 x by the 2 from the fraction

= 40%

40%--------------------------

Step-by-step explanation:

65 = 100%

0.65 = 1%

26 = x%

x = (26 ÷ 0.65)%

x = 40%

--------------------------FOLLOW ME

Two cars drive on the same highway in the same direction. The graphs show the distance, , of each one as a function of time, . Which car drives faster? Explain how you know.

Answers

Answer:

Car b

Step-by-step explanation: Because it’s going higher on the graph because it’s going more straight than car a which is going more sideways

Find the length of the shorter leg in the triangle pictured below

x=6
x=24
X=12
X=12 square root 3
X=36

Answers

Here is the answer X=36

Answer:

b. x=24

Step-by-step explanation:

There are special formulas for finding the sides of a 30-60-90 triangle. The hypotenuse is always equal to 2a and the long leg is always equal to [tex]a\sqrt{3}[/tex].

Since side a is given already as 12. We can substitute 12 for a.

2(12) = 24

An experiment consists of randomly selecting a marble from a bag, not replacing it, and then selecting another marble. The bag contains 3 yellow marbles and 2 white marbles. What is the probability of selecting a white marble and then a yellow marble?

Answers

Answer:

The bag contains 4 blue marbles, 3 green marbles, 7 red marbles, and 1 yellow marble. What is the probability of selecting a red marble and then a blue marble? Question 3 ... 4 blue marbles+ 3 green marbles+ 7 red marbles+ 1 yellow marble= 15 ... P(red,blue0 = 7/15 * 2/7 = 2/15 =.13333333 = 13.33%.

Step-by-step explanation:

Answer:

There are 14 total marbles. The probability of drawing a yellow marble on the first draw is 3/14. Given replacement, the probability on the second draw is the same as on the first draw, so the probability of a black marble on the second draw is 6/14. The probability of drawing first a yellow marble then a black marble is 3/14 * 6/14 = 18/196.

Angle ABD is 52°. State the size of angle DEB, giving a reason for your answer.

Answers

Answer:

∠ DEB = 52°

Step-by-step explanation:

The angle between a tangent and a chord is equal to the angle in the alternate segment, that is

∠ DEB = ∠ ABD = 52°

Answer:   angle DEB = 52 degrees

The short answer is that you use the tangent-chord theorem. A more lengthy answer is given below.

=================================================

Explanation:

Refer to the diagram below.

I'm going to move point E so that segment EB becomes a diameter of the circle (it goes through the center point F). Points A,B,C,D will stay in place. The red points and red segments show the changes.

In this slightly redrawn diagram, triangle EDB is a right triangle where angle EDB is 90 degrees. This is due to Thales Theorem, which is a special case of the inscribed angle theorem.

Also, the new segment EB is perpendicular to segment AC because the diameter touching the point of tangency is perpendicular to the tangent line. So angle EBA is 90 degrees.

Since points A, D and B didn't move, this means angle ADB is still 52 degrees. This makes the new angle EBD equal to 90-52 = 38 degrees.

Despite the fact that angle E has moved, it has not effected the size of angle DEB. This angle is the same as its original because we're still subtending the same minor arc DB, and that arc hasn't changed size (since segment DB hasn't changed). So that's why it's valid to use the same x for the old angle DEB in black and the new angle DEB in red. They're the same angle.

-------------

Focus on triangle EBD. We have the following interior angles

B = 38D = 90E = x

For any triangle, the three interior angles must add to 180

B+D+E = 180

38+90+x = 180

128+x = 180

x = 180-128

x = 52

Angle DEB is 52 degrees

It's not a coincidence that we end up with the same angle as shown in the original diagram. The tangent-chord theorem says that the angle between a chord and tangent (as shown in the diagram) is exactly equal to the inscribed angle that subtends the arc formed by the chord. Arc DB is formed by chord DB.

In short, the tangent-chord theorem says that inscribed angle DEB and angle ABD are the same measure (both are 52 degrees).

This demonstration I've shown is one way to prove the tangent-chord theorem. Though a more rigorous proof would involve another variable.

can you solve this problem for me 40/4=x/2

Answers

Answer:

40/4=x/2

10×2=x

x=20 is your answer

x = 20

is your answer

Let's solve your equation step-by-step.

40

(40)*(2)=x*(4)

80=4x

Step 2: Flip the equation.

4x=80

Step 3: Divide both sides by 4.

Other Questions
Find the marginal cost, marginal revenue, and marginal profit functions. C(x) = 7x; R(x) = 9x 0.001x2 alguien me ayuda con esto ? Johns brown attack on hapers ferry A large store has a warehouse it uses for storage. Trucks back up to the loading dock where merchandise is unloaded, sorted, and stacked in the correct area of the warehouse. The large shelves in the storage area are 17 feet 8 inches apart so the forklift machines can operate between the shelves. Is that distance greater than or less than 216 inches? All of the following are considered methodologies of art, except which?a.formalismc.imageryb.Feminismd.Marxismanswer: c. imagery Discuss the meaning behind the term, Desert of Death. what groups in texas did not want ex-confederates to have a voice in the texas government Please help me ASAPWhat is the area of this composite shape? the area of a rectangular lawn is 255 metre square if its length is 15 M find its perimeter the Gol Gumbaz is located at In Central Asia, farming is most common in the __________.A.river valleys of UzbekistanB.mountains of western ChinaC.plains of KazakhstanD.deserts of Turkmenistan i Put the answer for u! :) Kim was adopted as a baby and raised by loving parents in an enriched environment. Studies have shown that ________ Katie is 1.73 m tall. George is 7 cm shorter than Katie. How tall is George?Give your answer in m At a clothing store, 3 T-shirts and 8 hats cost $ 65. The cost of 2 shirts and 2 hats is $ 30. How much does each shirt and hat cost? Given that 1 = 0.72a) How much is 410 in ?b) What is the to exchange rate? Which of the following isneeded to determine velocity?A) Direction onlyB) Direction & timeC) Distance & directionD) Distance, time and directionDirection onlyDirection & timeDistance & DirectionO Distance, time and direction a worksheet can have a maximum of number of rows why are LGBT+ social movements important (UK)? Please help I don't understand this at all What is the kinetic energy of a 0.50 KG ball that is traveling at a speed of 40m/s